a book which cost 300 naira was sold for 240naira.what was the loss percentage?​

Answers

Answer 1

Answer:

20%

Step-by-step explanation:

Since the original price of the book is 300 naira...

100% = 300 naira

1% = 300 ÷ 100 = 3 naira

240 ÷ 3 = 80

Since 80% is the percentage of the price after the loss..

100 - 80 = 20%


Related Questions

-49 = 7i, what number is the i.

Answers

Answer:

-7

Step-by-step explanation:

divide by 7 on both sides and you get i = -7

Could someone help me with this? I have to double check I’m right thank you

Answers

Solving linear equations using matrices involves steps ranging from creating an augmented matrix, performing row operations, back substitution, and then interpreting results

To solve a system of linear equations using matrices:

Step 1: Write the system of linear equations in matrix form.

Represent the coefficients of the variables as a matrix (called the coefficient matrix), and the constants on the right side of the equations as another matrix (called the constant matrix).

Step 2: Create the augmented matrix.

Combine the coefficient matrix and the constant matrix into a single matrix by appending the constant matrix as an additional column. This combined matrix is called the augmented matrix.

Step 3: Perform row operations to achieve row-echelon form.

Use row operations (swapping rows, multiplying a row by a constant, or adding/subtracting rows) to manipulate the augmented matrix into row-echelon form. Row-echelon form has zeroes below the diagonal and non-zero elements on the diagonal.

Step 4: Perform back-substitution.

Starting from the last row of the row-echelon form matrix, solve for the variables using back-substitution. Substitute the values of the variables you find into the previous rows to determine the remaining variable values.

Step 5: Interpret the results.

Once you have solved all the variables, you have found the solution to the system of linear equations. If there are infinitely many solutions or no solutions, this will be indicated by the row-echelon form.

To know more about augmented matrix refer to

brainly.com/question/12994814

#SPJ1

A researcher performed an experiment with two groups. She found the difference of the means for each group. Then
she combined the groups, chose two new groups, and found the difference between the means of those groups. She
repeated this process 200 times. The normal distribution of the difference in the means she found is given below. How
great would the difference in means between the first two groups have to be in order to be considered significant?
At least 8
O At least 10
At least 9
At least 7

Answers

Answer:

Step-by-step explanation:

To determine the significance level for the difference in means between the first two groups, we need to refer to the provided normal distribution. However, you haven't provided the details or parameters of the normal distribution, such as the mean and standard deviation. Without this information, it is not possible to determine the exact significance level required.

In hypothesis testing, the significance level, typically denoted as α (alpha), is chosen by the researcher before conducting the experiment. It represents the threshold at which the researcher considers the results to be statistically significant. Commonly used significance levels are 0.05 (5%) or 0.01 (1%).

Please provide the necessary parameters or more information about the normal distribution to determine the specific significance level for the difference in means between the first two groups.

3.3.4√20+5-43 calculate ​

Answers

Here are the steps to calculate the expression √20 + 5 - 43 :

Step 1: Simplify the square root of 20:

[tex]\sqrt{20}[/tex]

Step 2: Calculate the value of the square root:

[tex]\sqrt{20} = 2\sqrt{5}[/tex]

Step 3: Substitute the value of the square root into the expression:

[tex]2\sqrt{5} + 5 - 43[/tex]

Step 4: Perform addition:

[tex]2\sqrt{5} + 5 = 2\sqrt{5} + \frac{5}{1} = \frac{2\sqrt{5} + 5}{1}[/tex]

Step 5: Perform subtraction:

[tex]\frac{2\sqrt{5} + 5}{1} - 43 = \frac{2\sqrt{5} + 5 - 43}{1}[/tex]

Step 6: Simplify the numerator:

[tex]\frac{2\sqrt{5} - 38}{1}[/tex]

Step 7: Simplify the expression:

[tex]2\sqrt{5} - 38[/tex]

Therefore, the calculation √20 + 5 - 43 simplifies to 2√5 - 38.

Simplify > (x)3(−x3y)2

−x9y2

x5y2

−x6y2

x9y2

Answers

The simplified expression of the expression (x)³(−x³y)² is x⁹y²

Simplyfing the expression using the common factor

From the question, we have the following parameters that can be used in our computation:

(x)3(−x3y)2

Express properly

So, we have

(x)³(−x³y)²

Open the brackets

(x)³(−x³y)² = x³ * x⁶y²

Multiply (x)³ and x⁶ in the expression

So, we have the following representation

(x)³(−x³y)² = x⁹y²

Hence, the simplified expression is x⁹y²

Read more about expression at

https://brainly.com/question/723406

#SPJ1

Identify the sequence graphed below and the average rate of change from n = 1 to n = 3.

coordinate plane showing the points 2, 10, point 3, 5, point 4, 2.5, and point 5, 1.25

a
an = 20(one half)n − 1; average rate of change is fifteen halves

b
an = 10(one half)n − 1; average rate of change is fifteen halves

c
an = 20(one half)n − 1; average rate of change is negative fifteen halves

d
an = 10(one half)n − 1; average rate of change is negative fifteen halves

Answers

The sequence based on the information would be: B. an = [tex]10 1/2^{-1}[/tex]average rate of change is negative fifteen halves.

How to explain the function

This question is about exponent function/series. You are given 3 points from the function, point A (1,10), point B( 2, 5), and point  C(4,1.25).  

If you insert point A to the function, an will give a result of 10 for n=1.

For n=0, the result would be:

an= 10([tex]1/2^{-1}[/tex])

= 10 * 2

= 20

Then the average rate of change from n=0 to n=2 would be:

Rate of change= (y₂ -y₁)/ (x₂ -x₁)

Rate of change= (5-20)/ (2-0)

= -15/2

= negative fifteen halves

Learn more about functions on

https://brainly.com/question/7717287

#SPJ1

100 Points! Geometry question. Photo attached. Please show as much work as possible. Thank you!

Answers

From the relationship of AC bar ≅ BD bar, we can tell that ABCD is a rectangle.

How is this a rectangle ?

A quadrilateral with two sets of parallel sides is known as a parallelogram.

If ABCD is a parallelogram where AC and BD have equal lengths, then the opposite sides of ABCD are also equal. ABCD has two sets of sides that are parallel and have the same length.

The fact that ABCD possesses two sets of sides that are parallel and of equal length logically leads to the conclusion that it also has four angles that are right. One reason for this is that the angles opposite each other in a parallelogram are of equal measure, while the sum of the angles at each corner of a parallelogram is always 180 degrees.Therefore, ABCD is a rectangle.

Find out more on rectangles at https://brainly.com/question/29782823

#SPJ1

solve for v, PV/T=pv/t

Answers

No matter the values of P, V, T, p, and t, as long as the equation PV/T = pv/t holds true, the solution for v is always equal to V.

To solve for v in the equation PV/T = pv/t, we can manipulate the equation to isolate v on one side.

Starting with the given equation:

PV/T = pv/t

To isolate v, we can cross-multiply:

PVt = pVt

Next, we divide both sides of the equation by pt:

V = v

Therefore, the solution is v = V.

In other words, v is equal to V, which means they represent the same values.

This implies that v and V are interchangeable and can be used interchangeably in the equation.

For similar question on equation.

https://brainly.com/question/17145398  

#SPJ11

some help is required

Answers

54° is the value of the given angle BAC.

As we know that the arc and angles formed by intersecting chords,

θ = (α + β)/2    .....(i)

According to the given figure, we have

θ = 10x+4

α =9x-12

β = 12x +15

Substitute the value in equation (1)

10x+4 = (9x-12+12x +15)/2

20x+8 = 21x +3

x = 8-3

x = 5

Thus,

∠BAC = θ

= 10*5+4

= 50 + 4

=54

Therefore, the value of the given angle will be 54°.

Learn more about the chord theorem here:

https://brainly.com/question/15298662

#SPJ1

During the winter, if the low temperature outside is a °C, the daily cost to heat a
building can be determined using the function f(x) = 5 (1.3) * . Find and
interpret the given function values and determine an appropriate domain for the function.
round all function values to the nearest hundredth.
f:7)= __, meaning when the low temp outside is __ celsius it would cost $__ to heat the building. this interpretation _______________ in the context of your problem.
f(9.5)=___, meaning when the low temp outside is ___ celsius it would cost & __ to heat the building. this interpretation _______________ in the context of the problem.
based on the observations above it is clear that an appropriate domain for the function is —————
PLEASE HELP

Answers

The values of the function are 5 and 1.5 and the domain is,

⇒ - ∞ < x < ∞

Here, The equation of the function is given as

f(x) = 5 (1.3)ˣ

The above function is an exponential function.

An exponential function is represented as

f(x) = abˣ

Where

a represents the initial value, b represents the rate of growth or decay

While x and f(x) are the input and output values

Hence, By comparison, we have

a = 5 and b = 1.3

This means that the function values are 5 and 1.3.

Since, the function is an exponential function.

The domain of an exponential function is the set of all real values

Hence, the domain of f(x) is the set of all real values.

Read more about exponential function at

brainly.com/question/11464095

#SPJ1

Write the equation of a line perpendicular to the one above that passes through (-2, 9). You may use either slope intercept or point slope form.

Answers

Answer:

-3x + 3

Step-by-step explanation:

To find the equation of a line perpendicular to the line passing through (-2, 1) and (4, 3), we need to determine the slope of the original line first. Then, we can use the negative reciprocal of that slope to find the slope of the perpendicular line. Finally, we can use the point-slope form to write the equation of the perpendicular line.

Step 1: Find the slope of the original line.

Slope (m) = (change in y) / (change in x)

m = (3 - 1) / (4 - (-2))

m = 2 / 6

m = 1/3

Step 2: Determine the slope of the perpendicular line.

The slope of the perpendicular line is the negative reciprocal of the original line's slope.

Perpendicular slope = -1 / (1/3)

Perpendicular slope = -3

Step 3: Use the point-slope form to write the equation.

The point-slope form is given by:

y - y1 = m(x - x1)

Using the point (-2, 9) and the perpendicular slope (-3), we can write the equation as:

y - 9 = -3(x - (-2))

y - 9 = -3(x + 2)

y - 9 = -3x - 6

y = -3x + 3

Therefore, the equation of the line perpendicular to the line passing through (-2, 1) and (4, 3) and passing through (-2, 9) is y = -3x + 3.

The product of a number and five added to 17

Answers

Answer:

The phrase "The product of a number and five added to 17" can be represented mathematically as:

5x + 17

In this expression, 'x' represents the unknown number. The product of the number and five is calculated by multiplying the number by 5, and then 17 is added to the result.

e
B
0
14. The table shows the number of inches of
rain over five months. What would be an
appropriate display of the data? Explain.
(Lesson 2)
Month
Number
of Inches
of Rain
Jan. Feb. Mar.
1.5
2.2
3.6
Apr.
5.3
May
4.8

Answers

The graph of the given function is attached.

Given is a function for the rainfall in 5 months in inches.

We need to display the data,

So, as we can see that the data is not showing any proportion or pattern,

So, it can be displayed as a line chart.

Hence the chart is attached for the function.

Learn more about line chart click;

https://brainly.com/question/29359235

#SPJ1

The algebraic expression below is a polynomial. x-1^k, where k is a real number

Answers

Answer: True

Step-by-step explanation:

50 Points! Multiple choice geometry question. Photo attached. Thank you!

Answers

Answer:

5/4 = h/44, so h = 55

The height of the tree is 55 feet.

D is correct.

Find the area of the composite figure

Answers

The area of the composite shape is 125units²

What is area of shape?

The area of a figure is the number of unit squares that cover the surface of a closed figure.

A composite shape can be defines as a shape created with two or more basic shapes.

The composite shape can be divided into 2 equal squares and a rectangles.

Area of the square = l²

= 5× 5 = 25

For two squares it will be;

25 × 2

= 50 units²

area of the rectangle = l× w

where w is the width

A = 15 × 5

= 75 units²

Therefore the area of the composite shape = 50 + 75 = 125 units²

learn more about area of composite shapes from

https://brainly.com/question/25965491

#SPJ1

Cameron surveyed her friends about the number of apps they use. The responses were 15, 16, 18, 9, 18, 4, 19, 20, 17, and 36 apps. Use the range and interquartile range to describe how the data vary.

Answers

The middle half of the data values vary by:

Interquartile = [tex]Q_3-Q_1[/tex] =>  19.25 - 13.5 = 5.25

The given data is :

15, 16, 18, 9, 18, 4, 19, 20, 17, and 36 apps.

We have to find:

Use the range and interquartile range to describe how the data vary.

We have to arrange the data from smallest to largest.

4, 9, 15, 16, 17, 18, 18, 19, 20, 36

Range of the data is : Highest no. - lowest no.

Range of the data is: 36 - 4 = 32

The location of the [tex]Q_1[/tex] = (n + 1) × 0.25 (where n is the total no. in the data)

The location of the [tex]Q_1[/tex] = (10 + 1)  × 0.25 = 11  × 0.25 = 2.75

∴[tex]Q_1[/tex] = 15 × 0.75 + 9 × 0.25  = 13.5

The location of the [tex]Q_3[/tex] = (n + 1) × 0.75

The location of the [tex]Q_3[/tex] = 11 × 0.75 = 8.75

∴[tex]Q_3[/tex] = 19 × 0.75 + 20 × 0.25 = 19.25

Hence, The middle half of the data values vary by:

Interquartile = [tex]Q_3-Q_1[/tex] =>  19.25 - 13.5 = 5.25

Learn more about Range at:

https://brainly.com/question/31016665

#SPJ1

b) Find the sum of all the numbers between 0 and 207 which are exactly divisible by 3.

Answers

27 and 9 are two numbers between 0 & 207 which are divisible with three and then their sum = 27+9=36

Will give you brainly! :) Thank you for taking time out of your day to answer this!​

Answers

The probability values are P(30 ≤ Amount ≤ 59) = 0.43, P(Amount ≥ 60 or Amount < 30) = 0.57 and P(Amount ≥ 40) = 0.45

Calculating the probability values

At least $30 but not more than $59

This means that we use the money spent from 30 to 59

So, we have

30 to 59 = 76 + 58 + 67

30 to 59 = 201

Total = 85 + 98 + 201 + 84

Total = 468

Next, we have

P(30 ≤ Amount ≤ 59) = 201/468

P(30 ≤ Amount ≤ 59) = 0.43

At least $60 or less than $30

This means that we use the money spent less than 30 and greater than or equal to 60

So, we have

At least $60 or less than $30 = 84 + 85 + 98

At least $60 or less than $30 = 267

Next, we have

P(Amount ≥ 60 or Amount < 30) = 267/468

P(Amount ≥ 60 or Amount < 30) = 0.57

At least $40

This means that we use the money spent greater than or equal to 40

So, we have

At least $40 = 58 + 67 + 84

At least $40 = 209

Next, we have

P(Amount ≥ 40) = 209/468

P(Amount ≥ 40) = 0.45

Read more about probability at

https://brainly.com/question/24756209

#SPJ1

................................................

Answers

Answer:

4,12,36,108,324.....

Step-by-step explanation:

The sequence represented by the equation [tex]a=4(3)^{n-1}[/tex]is a geometric sequence with first term 5 and common ratio 3. The first 4 terms of the sequence are:

[tex]a_1 = 4(3)^{1-1} =4(3)^0 = 4[/tex]

[tex]a_2 =4(3)^{2-1} = 4(3)^1 = 12[/tex]

[tex]a_3 =4(3)^{3-1} = 4(3)^2 = 36[/tex]

[tex]a_4 = 4(3)^{4-1} =4(3)^3 = 108[/tex]

[tex]a_5=4(3)^{5-1} =4(3)^4=324[/tex]

The sequence can be written as:

[tex]a_n = 4(3)^{n-1}[/tex]

where n is the term number.

Therefore, the sequence is 4,12,36,108,324.....

Answer:

The answer is 4,12,36,108,324

Step-by-step Explanation:

a1=4(3)¹‐¹=4(3)⁰=4

a2=4(3)²‐¹=4×3=12

a3=4(3)³‐¹=4(3)²=4×9=36

a4=4(3)⁴‐¹=4(3)³=4×27=108

a5=4(3)⁵‐¹=4(3)⁴=4×81=324

What are the solutions of the equation 2 log x = log (5x-6)? Select all that apply.
A x=1
B. x=2
C. x= 3
D. x = 5
E. x=6

Answers

Answer:

B, C

Step-by-step explanation:

x^2 = 5x - 6

x = 2 or x = 3

The length and breadth of a rectangular flower bed are 16m and 9 m, respectively. How many plants can be planted in it, if each plant requires a space of 1.2m x 1m?​

Answers

The calculated number of plants the flower bed can contain is 120

Calculating hw many plants can be planted in it

From the question, we have the following parameters that can be used in our computation:

Dimensions = 16 m by 9 m

So, the area of the flower bed is

Area = 16 * 9

Evaluate

Area =  144

Also, we have

Each plant requires a space of 1.2m x 1m?

This means that

Plant area = 1.2 * 1

Plant area = 1.2

So, we have

Plants = 144/1.2

Evaluate

Plants = 120

Hence, the number of plants is 120

Read more about area at

https://brainly.com/question/24487155

#SPJ1

Answer:

Step-by-step explanation:

To calculate the number of plants that can be planted in the rectangular flower bed, we need to calculate the area of the flower bed and divide it by the space required for each plant.

The area of the flower bed is calculated by multiplying its length and breadth.

So, the area of the flower bed is 16m x 9m = 144 sq.m.

Each plant requires a space of 1.2m x 1m = 1.2 sq.m.

Therefore, the number of plants that can be planted in the flower bed is:

144 sq.m. ÷ 1.2 sq.m./plant = 120 plants.

So, you can plant 120 plants in the rectangular flower bed.

Find area of the shape

Answers

Answer:

130.4

Step-by-step explanation:

6 x 14 = 84

8 x 4 = 32

(4 x 7.2)/2 = 14.4

A popular brand has introduced a new design of jeans. All major stores have stocked the jeans because the brand usually sells very well. But the cost of these jeans is much higher than those of other brands, so people don't buy them.

Answers

The price of the jeans will decrease because supply is greater than demand hence the stores will sell them at discounted prices.

How do we calculate?

The store owners would be forced to cut pricing if the jeans weren't purchased at all or as frequently as the other brands. Either they choose that choice, return the goods if possible  or just toss the jeans away. It generally takes some trial and error to determine the proper price.

For instance, the owner may lower the price to $75 and test whether the 25% discount works or not if the jeans initially sell for $100 and nobody buys them for a week.

The price might remain there if people continue to buy and if not, the cost might decrease even further.

Learn more about demand at:

https://brainly.com/question/1245771

#SPJ1

A kid's size small T-shirt is designed to fit children
who weigh between 43 and 55 pounds.
a. Write an inequality to describe w, the weight
of a child who has outgrown the small T-shirt.
9 √
b. Write an inequality to describe y, the weight
of a child who is not ready for the small T-shirt
yet.

Answers

A) The inequality that the weight of a child who has outgrown the small T-shirt is w > 55. B) The inequality that the weight of a child who is not ready for the small T-shirt yet. Is y < 43

How to determine the inequalities

a. To describe the weight (w) of a child who has outgrown the small T-shirt, we can use the inequality:

w > 55

This inequality states that the weight of a child (w) must be greater than 55 pounds for them to have outgrown the small T-shirt.

b. To describe the weight (y) of a child who is not ready for the small T-shirt yet, we can use the inequality:

y < 43

This inequality states that the weight of a child (y) must be less than 43 pounds for them to not be ready for the small T-shirt yet.

Learn more about inequality at https://brainly.com/question/25275758

#SPJ9

A telephone pole is 54 feet tall. A guy wire runs 83 feet, from point A at the top of the telephone pole, to the ground at point B. The base of the telephone pole is at point C. Triangle ABC is a right triangle.

How far from the base of the telephone pole, to the nearest tenth of a foot, is the guy wire secured to the ground at point B?

Answers

Okay, let's break this down step-by-step:

* The telephone pole is 54 feet tall

* The guy wire runs 83 feet from point A (top of pole) to point B (ground)

* So the hypotenuse (AB) of the right triangle is 83 feet

* The opposite side (AC) is 54 feet (height of pole)

To find the adjacent side (BC), we use the Pythagorean theorem:

a^2 + b^2 = c^2

54^2 + BC^2 = 83^2

Solving for BC gives:

BC = sqrt(83^2 - 54^2) = sqrt(1296 - 2916) = sqrt(1620) = 40 feet

So the guy wire is secured 40 feet from the base of the telephone pole.

Rounded to the nearest tenth is 40.0 feet.

Therefore, the final answer is:

40.0

Let me know if you have any other questions!

Find the Value of x. Round to the nearest tenth.

hyp = x
28°
11​

Answers

The calculated value of x in the right triangle is 12.5

How to calculate the value of x

From the question, we have the following parameters that can be used in our computation:

The right triangle

The value of x can be calculated using the following cosine rule

So, we have

cos(28) = 11/x

make x the subject of the formula

So, we have

x = 11/cos(28)

Evaluate the quotient

This gives

x = 12.5

Hence, the value of x is 12.5

Read more about right triangles at

https://brainly.com/question/2437195

#SPJ1

Please help due 5 min!!!!

Answers

The midsegments of a trapezoid is parallel to each base, and its length is one-half the sum of the lengths of the bases

b) tan 2A cot A-1 = sec 2A

Answers

The trigonometric equation is tan 2A cot A-1 = sec 2A

We have to prove the trigonometric equation

tan 2A cot A-1 = sec 2A

Now let us take LHS

tan 2A cot A-1  

2tanA/1-tan²A - 1 . cotA - 1

2-1+tan²A/1-tan²A

1+tan²A/1-tan²A

sec2A

Hence, the trigonometric equation is tan 2A cot A-1 = sec 2A

To learn more on trigonometry click:

https://brainly.com/question/25122835

#SPJ1

5. [0.5/1 Points] DETAILS PREVIOUS ANSWERS SALGTRIG4 7.3.112.
A rectangle is to be inscribed in a semicircle of radius 4 cm as shown in the following figure.
10
4 cm-
(a) Find the function that models the area of the rectangle.
A(0) = 16 sin (20)
Need Help? Read It
MY NOTES
4
ASK YOUR TEACHER
(b) Find the largest possible area for such an inscribed rectangle. [Hint: Use the fact that sin(u) achieves its maximum value at u=/2.]
16
cm²
(c) Find the dimensions of the inscribed rectangle with the largest possible area. (Round your answers to two decimal places.)
smaller dimension 2
x cm
larger dimension 25
x cm
PR.

Answers

The Area function [tex]A(x) = 2x * \sqrt(16 - x^2).[/tex]

How to solve

(a) For a rectangle inscribed in a semicircle of radius 4 cm, let half-length be x and height be y.

Therefore, the Area function [tex]A(x) = 2x * \sqrt(16 - x^2).[/tex]

(b) Maximum area occurs when x = [tex]4 * sin(\pi/4), so A(4 * sin(\pi/4)) = 16 cm^2.[/tex]

(c) Dimensions for the largest inscribed rectangle are smaller dimension (height) ≈ of 2.83 cm, a larger dimension (base) ≈ of 5.66 cm.

Read more about area here:

https://brainly.com/question/25292087

#SPJ1

Other Questions
2 questions pls help it would mean everything to me According to the current structure of interest rates, the effective annual interest rates for 1, 2 and 3 year maturity zero coupon bonds are 81 = 0.08 $2 = 0.10, 83 =0.11. Find the one-year forward effective annual rate of interest and find the two-year forward effective annual rate of interest. for the sn2 reactions, you can see a difference in leaving groups when comparing the rate of reaction of bromobutane and which other alkyl halide? 1-chlorobutane which is the better leaving group? N a survey of 1000 U. S. Teenagers, 41% consider entrepreneurship as a career option. The margin of error is $\pm3. 2$ %. A. Give an interval that is likely to contain the exact percent of U. S. Teenagers who consider entrepreneurship as a career option. Between % and %Question 2b. The population of teenagers in the U. S. Is about 21. 05 million. Estimate the number of teenagers in the U. S. Who consider entrepreneurship as a career option. 1. Protective sacs (valves )2. Carries blood to the body (pulmonary) 3. Carries blood to the lungs (heart chambers) 4. Open and close (pericardium) 5. Atria and ventricles (aorta) Which of the following is an example of discrimination? A. Luke thinks all women are manipulative. B. Ryan feels mistrustful of Jews Return on Assets The following information was taken from recent Apple Inc.'s financial statements. Numbers are in millions. 2017 2016 Net Income $48,351 $45,687 Total Assets 375,319 321,686 Required a. What was Apple's return on assets for 2017 and 2016? (Round to one decimal point) b. Based on your answer from part a., how did the company's performance change from 2016 to 2017? a. 2017 2016 0 % Return on Assets 096 b. Apples's performance, as measured by its return on assets, from 2016 to 2017. Use the method of iteration to find a formula expressing sn as a function of n for the given recurrence relation and initial conditions. S(n) = 5 - 3S(n-1), S(0) = 2 Lucas is a business manager for Micro Manufacturing Company. Ethical dilemmas that Lucas is not likely to encounterinclude deciding A. the kind of pizza to order for a company meeting. B. whether to lay off employees or take a loss in corporate profits.C. whether to admonish an employee making rude jokes in the workplace.D .what to tell a reporter about the recent drop in corporate stock prices. You have a goal of having $300,000 four years from today. The return on the Investment is expected to be 8% and will be compounded seml-annually. The amount that needs to be Invested today Is closest to: (FV of $1. PY of $1. FVA of $1, and PVA of $1 (Use appropriate factor(s) from the tables provided.) O $150000 O $219,210 O $200,000. O $219,910 as of today, almost all of the potatoes used for mcdonald's french fries in india are imported.TRUE OR FALSE if it is found that peoples self-esteem changes quicklyfor instance, from day to daythen self-esteem should be considered to be given r=(x,y,z), s=(u,v,w,t) the following is a valid relational algebra expression: Discuss technologies and Revolution by means of production, consumption and regulation a bond currently has a price of $1,050. the yield on the bond is 6%. if the yield increases 25 basis points,the price of the bond will go down to $1,030. compute the duration of this bond in years. Three forces act on the bracket Determine the reactions at the gound from these 3 forces Problem Data: F = 125 F2 = 139 F3 = 145 . d = 5.9 d. - 8.4 ds = 8.6 NOTE Enter numerical values only! Graded as: Correct answers are within 4% of solutions . . . 3. Reaction at the gound in x: R b. Reaction at the gound in y Ry = c. Moment at the gound in Musing the sign convention in the drawing : M = in Ali has X paintings He buys two more paintings.How many Paintings does he now have? true or false: a country's trade deficit often increases during periods of currency appreciation. true false Natasha places an online order for plate holders to display her antique plates. She chooses a specific site because it has a promotional offer of 15% off on all purchases. She orders 3 large holders for $4. 95 each, 2 medium holders for 3. 25 each and 2 small holders for $1. 75 each. There is no sales tax on her purchase, but she must pay a flat rate of $5. 35 for shipping and handling. What is the total of Natasha?s online purchase? a. $19. 64 b. $24. 18 c. $26. 47 d. $28. 45 Please select the best answer from the choices provided A B C D. true/false. it is important that the practitioner base any practice schedule modification on performance difficulties evident from practice sessions rather than on those from retention or transfer tests.